2
$\begingroup$

This question is a twist on a question I asked here Random pseudo-walk of Poisson variables, but with randomly 'disappearing' objects. I do not know how to generalize the (satisfactory) answer given there.

Suppose there is a pool that can contain any non-negative number of objects. At time 𝑑 it contains $n_t$ objects. Time is discrete.

Before time 𝑑+1 three things happen, in this order:

  1. Unless the pool is empty, one object is removed from it.
  2. Each of the remaining $n_t-1$ objects independently disappears from the pool, with probability $\delta$. So we remain with $m_t \sim Bin(n_t-1, 1 - \delta)$ objects.
  3. A number of objects π‘žβˆΌPoisson(πœ†) are added to the pool, where 0<πœ†<1. π‘ž is drawn from a Poisson distribution with expectation πœ†, independently of previous draws. So $𝑛_{𝑑+1}=m_𝑑+π‘žβˆ’1$, unless $𝑛_𝑑=0$, in which case $𝑛_{𝑑+1}=π‘ž$.

Let $𝑛_0=0$. What is the distribution of $𝑛_𝑇$, for any large 𝑇? Let's call it $𝑛_\infty$. What is $𝑛_\infty$'s expectation, as a function of πœ†?

With $\lambda>1$, does $𝑛_\infty$ necessarily diverge to +∞, as it did without disappearances?

This is my motivation: Objects are bidders in a recurring auction, with values i.i.d. drawn from a distribution $F(x)$, and whose values exceed some $y > 0$. The removed object is an auction winner, who leaves. In this question, the bidders discount their values each round, so with some probability fall below $y$.

$\endgroup$
0

1 Answer 1

1
$\begingroup$

Here's a partial answer, that I hope someone can complete.

Assuming there is a stationary state, let $X$ be a random variable that is the number of objects. Let $p_n := \Pr[X = n]$.

Let $G(z) := \sum_{n=0}^\infty p_n z^n$ be the probability generating function for $X$. After removing one object, its PGF is $$ p_0 + p_1 + \sum_{n=2}^\infty p_n z^{n-1}, $$ and after each remaining element disappears with probability $\delta$, its PGF is $$ p_0 + p_1 + \sum_{n=2}^\infty p_n [\delta + (1-\delta)z]^{n-1}. $$ The Poisson arrivals have PGF $\phi(z) = e^{\lambda(z-1)}$. Their distribution is independent of the previous distribution, so by the law for independent distributions, the resultant PGF is their PGFs multiplied $$ \Big\{p_0 + p_1 + \sum_{n=2}^\infty p_n [\delta + (1-\delta)z]^{n-1}\Big\}\phi(z). $$ Finally, since this probability is stationary, this equals the original probability, i.e., $$ G(z) = \Big\{p_0 + p_1 + \sum_{n=2}^\infty p_n [\delta + (1-\delta)z]^{n-1}\Big\}\phi(z). $$ This is an implicit equation for $G(z)$. If it can be solved, analytically or numerically, we are done. This eludes me, so far.

$\endgroup$

You must log in to answer this question.

Start asking to get answers

Find the answer to your question by asking.

Ask question

Explore related questions

See similar questions with these tags.